Simplifying Algebraic Expressions with Multiple Variables

  • Thread starter Thread starter Natasha1
  • Start date Start date
  • Tags Tags
    Algebra
Click For Summary
The discussion focuses on simplifying the algebraic expression (3n+2)(3m+2)(3p+2). Participants confirm that the correct expanded form is 27mnp + 18(mn+mp+np) + 12(m+n+p) + 8, acknowledging that the expression is not particularly tidy. One user suggests a neater way to simplify the first two terms, resulting in 9nm + 6(n+m) + 4, but notes that the overall problem remains complex. The conversation emphasizes the importance of careful multiplication and organization in algebraic expressions. Ultimately, the participants agree on the correct expanded form, despite its complexity.
Natasha1
Messages
494
Reaction score
9
Sorry had a long day... just need this checking over please

(3n+2)(3m+2)(3p+2) = 27nmp + 6(m+n+p) + 8 :zzz:
 
Physics news on Phys.org
Not what I got. Try multiplying the first two terms, then multiplying the result by the third term. There is something missing in the middle.
 
Jeff Ford said:
Not what I got. Try multiplying the first two terms, then multiplying the result by the third term. There is something missing in the middle.

I get this

27 nmp+18p(m+n)+12p+18nm+12(m+n)+8

but isn't there a nicer way to simplify it like (3n+2)(3m+2)= 9nm+6(n+m)+4
which is all neat and tidy?
 
Last edited:
It's not a particularly tidy problem. I got the same answer you did, but wrote it like this

27mnp + 18(mn+mp+np) + 12(m+n+m) + 8

That's about as pretty as it gets.
 
Jeff Ford said:
It's not a particularly tidy problem. I got the same answer you did, but wrote it like this

27mnp + 18(mn+mp+np) + 12(m+n+m) + 8

That's about as pretty as it gets.

Thanks sunshine! you must have meant 27mnp + 18(mn+mp+np) + 12(m+n+p) + 8 :-)
 
Question: A clock's minute hand has length 4 and its hour hand has length 3. What is the distance between the tips at the moment when it is increasing most rapidly?(Putnam Exam Question) Answer: Making assumption that both the hands moves at constant angular velocities, the answer is ## \sqrt{7} .## But don't you think this assumption is somewhat doubtful and wrong?

Similar threads

  • · Replies 12 ·
Replies
12
Views
3K
  • · Replies 10 ·
Replies
10
Views
2K
  • · Replies 2 ·
Replies
2
Views
2K
  • · Replies 4 ·
Replies
4
Views
2K
Replies
5
Views
3K
  • · Replies 5 ·
Replies
5
Views
3K
  • · Replies 13 ·
Replies
13
Views
2K
  • · Replies 1 ·
Replies
1
Views
1K
  • · Replies 8 ·
Replies
8
Views
2K
  • · Replies 6 ·
Replies
6
Views
4K